LSAT and Law School Admissions Forum

Get expert LSAT preparation and law school admissions advice from PowerScore Test Preparation.

User avatar
 Dave Killoran
PowerScore Staff
  • PowerScore Staff
  • Posts: 5853
  • Joined: Mar 25, 2011
|
#87414
Complete Question Explanation
(The complete setup for this game can be found here: lsat/viewtopic.php?f=149&t=16450)

The correct answer choice is (E)

This is the first of two Justify questions in this game. This occurrence is unusual because no prior game has ever featured two Justify questions.

In this instance, you are asked to identify the piece of information that will completely determine when each monument was begun. In order to achieve this end, you must find an answer that places a monument in such a way that other monuments are immediately effected, and consequently limited so that each can only have been begun in a single year.

Answer choice (A): Placing F in 603 forces G into 601 and L into 602, but there are still several options available for H, M, and S. Thus, this answer does not fix every monument and can therefore be eliminated.

Answer choice (B): This answer looks promising because it takes a member of the G/M/S trio and removes it from 601, forcing the other two variables into 601. When G is placed in 602, the immediate result is that M and S must be in 601. With 601 and 602 filled, L must be in 603. Only F and S remain, but they can rotate between 604 and 605, and so this answer can be eliminated.

Answer choice (C): Placing H in 605 leaves a variety of options for the remaining variables, and so this answer can be eliminated.

Answer choice (D): This answer is also promising because it addresses a member of the G/M/S trio. However, like answer choice (B), the variable is placed into 602, leading to the same result as in (B). When M is placed in 602, the immediate result is that G and S must be in 601. With 601 and 602 filled, L must be in 603. Only F and S remain, but they can rotate between 604 and 605, and so this answer can be eliminated.

Answer choice (E): This is the correct answer. Like answer choices (B) and (D), a member of the G/M/S trio is taken out of 601, and so this answer is initially attractive. When S is begun in 604, G and M must be begun in 601. Because 604 is occupied, H must have been begun in 605. Only F and L remain, and due to the action of the first rule L must have been begun in 602 and F must have been begun in 603:

PT58-Sept2009 LGE-G1-Q3-D1.png
You do not have the required permissions to view the files attached to this post.
User avatar
 SGD2021
  • Posts: 72
  • Joined: Nov 01, 2021
|
#94444
Is the quickest way to answer this question to test out every answer choice?
 Rachael Wilkenfeld
PowerScore Staff
  • PowerScore Staff
  • Posts: 1358
  • Joined: Dec 15, 2011
|
#94452
Hi SGD,

I'd start with the area that gives you the most information. When I look at the overall set up, I think that the GMS in as two of the spots in 1 is the area I'd focus in. So I'm looking for something that impacts what is built in 601.

That narrows my options to answer choices B, D, and E.

For answer choice (B) when I put G is built in 602, M and S are built in 601. But I still can't quite place FLH. H could still be in either 4 or 5.

In answer choice (D), we structurally have the same thing as answer choice (B). When M is in 602, G and S are in 601. I still can't quite place FLH.

Answer choice (E) is the answer choice that provides the correct amount of restriction. When S is in 604, G and M are in 601. H can ONLY be in 605, because it's only in 604 and 605, and 604 is now taken by S. That leaves F and L for 2 and 3, which we know have to be in a specific order.

For what it's worth, I wouldn't eliminate answer choices (A) and (C) just based on the variables used, but I would leave them for the end of my attack. Once I saw that answer choice (E) was correct, I would pick it and move forward.

Hope that helps!

Get the most out of your LSAT Prep Plus subscription.

Analyze and track your performance with our Testing and Analytics Package.